LSAT and Law School Admissions Forum

Get expert LSAT preparation and law school admissions advice from PowerScore Test Preparation.

User avatar
 Dave Killoran
PowerScore Staff
  • PowerScore Staff
  • Posts: 5849
  • Joined: Mar 25, 2011
|
#44052
Complete Question Explanation
(The complete setup for this game can be found here: lsat/viewtopic.php?t=16041)

The correct answer choice is (A)

This is a “5 if” question, which is a type that can be quite time-consuming. Thus, you must use as many tricks as possible to narrow down the list of possible answer choices.

For example, the correct answer choice from question #12 can be used to eliminate answer choices (C) and (E). Since List questions such as #12 always produce full or partial hypotheticals, you can often use the correct answer in a List question to eliminate answer choices in other questions, as is the case here. List questions, which are among the easiest questions, are thus also among the most useful questions, and you must take special care to answer them correctly. By quickly and easily eliminating two answers on this difficult question you can save valuable time.

Answer choice (B) is unlikely to be correct because it posits that R, a random must be selected.

Answer choice (D) can be disproven by the hypothetical J-K-M-R-S.

Accordingly, answer choice (A) is correct.
 rameday
  • Posts: 94
  • Joined: May 07, 2014
|
#15568
So I understand how they eliminated answer C,B,E. But i don't understand how you would eliminate answer choice D & why B MBT.

A
User avatar
 KelseyWoods
PowerScore Staff
  • PowerScore Staff
  • Posts: 1079
  • Joined: Jun 26, 2013
|
#15584
Hi A!

In this question, we know that the chairperson is a homeowner which means we must have 2 homeowners and 3 tenants.

To eliminate answer choice (D), try to show that you can have J without Q while still having 2 homeowners and 3 tenants. You could have J M K R S. So, therefore, it does not have to be true that if you have J, you have Q.

Answer choice (A) must be correct because if you have G, you must have K and you can't have M or J. Since you must have 3 tenants, the only tenant left is F. If you have F, you must have Q. So, therefore, it must be true that if you have 2 homeowners and 3 tenants and you have G, you must also have Q.

Hope this helps!

Best,
Kelsey
 dandelionsroar
  • Posts: 27
  • Joined: Oct 18, 2018
|
#71997
Hi could you eliminate B and E because R and S are interchangeably variables?
 Zach Foreman
PowerScore Staff
  • PowerScore Staff
  • Posts: 91
  • Joined: Apr 11, 2019
|
#72032
If you mean that if R is an option then S is an option too, then no. It is possible that both R and S must be selected but they only mention R, so you can't eliminate based just on that.
But, I would deprioritize those two answers because R and S are "random" variables, i.e. they have no rules attached to them. While it is possible, it is unlikely that they would be forced to be or not be in a particular arrangement because they are not forced by any rules on them.
This is why it is important to identify the randoms at the start. I would approach this question by noting that A and B start with "If G" and the rest start with "If J..." and they all have a positive necessary condition "...is selected". I already know that there will be three tenants and two homeowners on the committee. There's no compelling reason to try G or J first. I might do J because there are three answers that start with it. So I get J which gives me M which eliminates P. So I have two out of three tenants and one out of two homeowners. I know I can't have G because then I would have to have K too and that would be four tenants. But I could have but wouldn't need F, Q or R so I eliminate those three.
Now let's see what G does. G gives me K and that seems like all. But since we know that we have exactly three tenants and we have 2 of them with G and K, we cannot have either M or J since that would be 4. So we need F. And if F then Q so our answer is A.
 sarah_tucker@alumni.brown.edu
  • Posts: 17
  • Joined: Sep 20, 2021
|
#99031
Hi,
After reading the posts above, I still do not understand what the question was asking. Is is addressing a hypothetical or a general inference from the game? Can you break down how to approach this question? Thanks!
 Adam Tyson
PowerScore Staff
  • PowerScore Staff
  • Posts: 5153
  • Joined: Apr 14, 2011
|
#99033
It's what we call a Local question, Sarah, because it starts with a restriction that applies to this question but which is not a general rule for the game. The restriction in this case is that the chairperson is a homeowner, which means we must be looking at a solution that has two homeowners and three tenants.

Then, within that scenario, we have to determine which of the conditional answers must be true. For example, answer A: when the chairperson is a homeowner, if G is selected as part of the committee, must Q also be selected? If so, that is the correct answer, and if not, then that answer is incorrect.

We have several strategies to apply to this question. Dave suggested one in the original explanation, which is to use your solutions to prior questions to help eliminate wrong answers. Look for any diagrams you have already drawn that had two homeowners and three tenants and see which answer choices did not have to be true, and cross them out. That way, if you have to test answer choices, you won't have to test as many. Kelsey and Zach described other strategies that involve testing options to see what they do and do not require.

I'll suggest another, and it goes back to the original setup of the game. I suggest setting up templates, one of my favorite approaches to many games. In this case, with a game that has 8 questions, templates can often be worth the time and effort it takes to create them, especially since in this case there are only 4. Here's how I would do it:

Template 1: 2 homeowners and 3 tenants, with J and M included. G and P would be knocked out, so the remaining tenant would have to be either F or K, and the homeowners would be any pair from Q, R, and S. If F is the third tenant, Q must be one of the homeowners.

(QRS) JM F/K

Template 2: 2 homeowners and 3 tenants, with J and M excluded. That makes the tenants FGK, one of the homeowners must be Q, and the other homeowner could be any of P, R, and S.

(Q, P/R/S)FGK

Template 3: 2 tenants and 3 homeowners, with J and M included. P is knocked out, so the homeowners must be QRS.

JMQRS

Template 4: 2 tenants and 3 homeowners, with J and M excluded. The tenants must include K with either F or G, and the homeowners can be any group of 3. Remember that if F is selected, Q must also be selected.

F/G K (PQRS)

Using these templates, the local restriction forces us to focus only on the first two templates, and answer A turns out to be true. There is only one template with 2 homeowners and G as one of the tenants, and Q is required in that group.

B does not have to be true because R is optional in that template that includes G.

The remaining answers do not have to be true, because in the template that has J and M among the three tenants, none of F, Q, or R are required.
 CammeronJ
  • Posts: 7
  • Joined: May 23, 2023
|
#102383
I read this question in reverse, I guess?

I thought it was asking what condition would force the chairperson to be a homeowner and went with ‘C’.

How do we know that this is a local instead of a global question?
 Adam Tyson
PowerScore Staff
  • PowerScore Staff
  • Posts: 5153
  • Joined: Apr 14, 2011
|
#102385
As soon as you see that "if" at the beginning of the question, you know you're in a local scenario, because it's introducing something that doesn't have to happen. It's just "what if this happens?" That immediately narrows down the circumstances under which the answers will be analyzed.

Get the most out of your LSAT Prep Plus subscription.

Analyze and track your performance with our Testing and Analytics Package.